Do the two trapezoids in the figure appear to be similar? Why or why not?
options:

A)

They're not similar because only one pair of corresponding angles in the two trapezoids is congruent.

B)

They're similar because two pairs of corresponding angles in the two trapezoids are congruent.

C)

They're similar because one pair of corresponding angles in the two trapezoids is congruent.

D)

They're not similar because two pairs of corresponding angles in the two trapezoids are congruent.

Do The Two Trapezoids In The Figure Appear To Be Similar? Why Or Why Not?options:A) They're Not Similar

Answers

Answer 1

Answer:

Option B,

They're similar because two pairs of corresponding angles in the two trapezoids are congruent

Answer 2

Answer:

They're not similar because only one pair of corresponding angles in the two trapezoids is congruent.

Step-by-step explanation:


Related Questions

Write the sum using summation notation, assuming the suggested pattern continues. 6, -18, 54, -162, +… Is this sequence arithmetic or geometric? Explain your answer.

Answers

Answer:

Hello,

This sequence is geometric with a ratio of -3

the first term is 6

Step-by-step explanation:

[tex]u_1=6\\u_2=-18=6*(-3)=u_1*(-3)\\u_3=54=-18*(-3)=u_2*(-3)=u_1*(-3)^2\\u_4=-162=u_3*(-3)=u_1*(-3)^3\\\\...\\u_{n+1}=u_1*(-3)^n\\\\\displaystyle \sum\limits^\infty _{i=1}u_i = \lim_{n \to \infty} \sum\limits^n _{i=1}u_1*(-3)^{i-1}\\=6*\lim_{n \to \infty} \sum\limits^\infty _{i=1}(-3)^{i-1}\\=6*\frac{1-(-3)^n}{1-(-3)} \\=\dfrac{3}{2} *({1-(-3)^n)\\[/tex]

serie does not converge.

Question 1 (Multiple Choice Worth 4 points)
(08.01) Looking at the spread of your data best fits which step of the statistical process?

Answers

Answer:

The answer is "Analysis the information by chart and number processes".

Step-by-step explanation:

They already have articulated a query and also gathered information unless you are searching only at the distribution of your results. Those who are ready to analyze your results for all are there.

i will rate you brainliest// What is the interquartile range (IQR) of {5.8, 8.5, 9.9, -0.8, -1.3, 2.3, 7.4, -1.9}?

Answers

Answer

arrange the element in increasing order

-1.9, -1.3, -0.8, 2.3, 5.8, 7.4, 8.5, 9.9

interquatile = Q3 - Q1

[tex] = \frac{7.4 + 8.5}{2} - \frac{ - 1.3 - 0.8}{2} [/tex]

[tex] = 7.95 + 1.05[/tex]

[tex] = 9[/tex]

Answer:

9.0

Step-by-step explanation:

i took the quiz

9x-5
14x+24
A
Determine the value of x.
1) x= -5.8
2) x= -7
3) x= 7
4) x= 32.2

Answers

Answer:

the answer to the question

is x=-5.8

Help me please…………………….

Answers

Answer:

8.7

Step-by-step explanation:

If you like my answer mark me brainliest

Find the next term of the sequence.
16, 9, 2, -5,

Answers

Answer:  The next term is -12.

Step-by-step explanation:

16,9,2,-5  

Looking at these numbers to go from 16 to 9 you will add -7 or subtract 7 . The same way you subtract 7 from 9 to get 2  and subtract 7 from 2 to get -5.

So to determine the next term subtract 7 from -7   or add -7.

-5 - 7 = -12

0r  -5 + -7 = -12  

[tex] &#128075 [/tex] Hello ! ☺️

Step-by-step explanation:

Find the next term of the sequence.

Let us find the interval between two successive terms:

16 - 9= 7

-7 is therefore the common différence of this sequence. (d)

Find the next term :

-5 + (-7)= -12

[tex]\boxed{\color{gold}{N = -12}} [/tex]

[tex]<marquee direction="left" scrollamount="2" height="100" width="150">&#x1F498;Mynea04</marquee>[/tex]

square root of 49/64 answered as a fraction

Answers

Answer:

Hey there!

That would be 7/8

Let me know if this helps :)

For his phone service, Ivan pays a monthly fee of $25, and he pays an additional $0.07 per minute of use. The least he has been charged in a month is $103.82.
What are the possible numbers of minutes he has used his phone in a month?
Use m for the number of minutes, and solve your inequality for m.

Answers

Answer:

25+(0.07xm)=103.82      m=1126

To measure Monarch butterfly migration,
scientist tag and release 100 butterflies in
Kansas and Missouri before traveling to
Mexico. While in Mexico, the same
scientist captured another 100 butterflies
of which 15 are tagged. Based on this
information, how many butterflies would
you predict start in Kansas and Missouri
and migrate to Mexico?

Answers

the answer is 667.

100/x = 15/100

it’s a proportion so you cross multiply. after cross multiplying you get 15x = 10,000. divided both sides by 15 and you get x = 66.6 (continuation of the 6). since it’s repeating, round it and you get 667.

by the way this is actually one of my homework questions for today ;)

The predicted number of butterflies that starts from Kansas and Missouri and migrate to Mexico is 667.

The Aim of the experiment is to measure the Monarch butterfly migration

number of butterflies tagged and released in Kansas and Missouri = 100 number of tagged butterflies captured in Mexico = 15 Let the number of butterflies that start from Kansas and Missouri and end in Mexico = x

Predicting the number of butterflies

= [tex]\frac{100}{x} = \frac{15}{100}[/tex]

= [tex]15x = ( 100 * 100)[/tex]

∴ x( predicted value ) = [tex]\frac{10000}{15 } = 667[/tex]

hence the approximate value of the number of butterflies that migrate to Mexico from Kansas and Missouri is  667

learn more about migration : https://brainly.com/question/23903333

BRAINLIEST ANSWER GIVEN, WHY CAN'T ANYONE HELP ME?! Find the equation of the line passing through the pair points (-8,6) (-9,-9). The equation of the line in the form is Ax+By=C.

Answers

Answer:

15x - y = - 126

or y = 15x + 126

Step-by-step explanation:

will make it simple and short

to find the equation... we need to find slope first.

                   y2 - y1             -9  -   6

slope = m  = ---------    =       -----------  =  15

                   x2 - x1             -9  -  (-8)

so we know that the equation of the line using point (-8,6) and slope 15             y - 6 = 15( x + 8)

y - 6 = 15x + 120

Writing the equation in the form   Ax + By = C

15x - y = -120-6

therefore.... 15x - y = - 126   or simplify it as or y = 15x + 126

Hope this helps

Combine like terms. What is a simpler form of each expression? 4c-4d+8c-3d

Answers

Answer:

12c-7d

Step-by-step explanation:

[tex]4c-4d+8c-3d=0\\4c+8c=3d+4d\\12c=7d\\12c-7d[/tex]

Answer: 12c-7d

===============================================

Explanation:

The terms 4c and 8c are one pair of like terms that combine to 4c+8c = 12c. We add 4 and 8 to get 12, then tack a 'c' at the end

The other pair of like terms are -4d and -3d. They combine to -7d for similar reasoning.

12c and -7d are not like terms, so we can't combine them and we stop here.

-----------

One way to think of combining like terms is consider simplifying 2c+3c. You could say that 2c represents having 2 cups while 3c is having 3 cups. Writing 2c+3c means we start with 2 cups and add on 3 more getting a total of 2+3 = 5 cups. Symbolically we would then write 5c. Therefore 2c+3c = 5c.

Is {(4,2),(4,-2),(9,3),(9,-3)} a function

Answers

Answer:

  no

Step-by-step explanation:

If any x-value is repeated, the relation is not a function. Both x=4 and x=9 are repeated values, so this relation is not a function.

Is the following relation a function? (1 point) x y −1 −2 2 3 3 1 6 −2 No Yes

Answers

Answer:

Yes because no same x-value resulted in different y-values.

Answer:

Yes

Step-by-step explanation:

The energy E (in ergs) released by an earthquake is approximated by log E= 11.8 + 1.5M. Where M is the magnitude of the earthquake. What is the energy released by the 1906 San Francisco quake, which measured 8.3 on the Richter scale? This energy, it is estimated, would be sufficient to provide the entire world's food requirements for a day. Answer in ergs.

Answers

Answer:

[tex]\large \boxed{3.4 \times 10^{10}\text{ ergs }}[/tex]

Step-by-step explanation:

[tex]\begin{array}{rcl}\log E & = & 11.8 + 1.5M\\& = & 11.8 + 1.5 \times 8.3\\& = & 11.8 + 12.45\\& = & 24.25\\E & = & e^{24.25}\\& = & \mathbf{3.4 \times 10^{10}} \textbf{ ergs}\\\end{array}\\\text{ The energy released was $\large \boxed{\mathbf{3.4 \times 10^{10}}\textbf{ ergs }}$}[/tex]

Find the value of x.
A. 3
B. 9
C. 0
D. 12

Answers

Answer:

x=3

Step-by-step explanation:

(segment piece) x (segment piece) =   (segment piece) x (segment piece)

3x(x+1) = 4x(x)

Divide each side by x

3x(x+1)/x = 4x(x)/x

3(x+1) = 4x

Distribute

3x+3 = 4x

Subtract 3x

3x+3-3x= 4x-3x

3 =x

Answer:

x = 3

Step-by-step explanation:

0 is a rediculas answer 9 and 12 are to big.

The lines are supposed to have a simular length:

3(3) + 4 = 13

4(3) + 3 = 15

These are the best answers that fit.

An equation for the depreciation of a car is given by y=A(1-r)t where y=current value of the car.A=original cost r=rate of depreciation and t=time in years. The value of a car is half what it originally cost. The rate of depreciation is 10%. Approximately how old is the car?

Answers

Answer: Approximately 6.58 years old

The more accurate value is 6.57881347896059, which you can round however you need. I picked two decimal places.

==================================================

Explanation:

Let's pick a starting value of the car. It doesn't matter what the starting value, but it might help make the problem easier. Let's say A = 1000. Half of that is 1000/2 = 500.

So we want to find out how long it takes for the car's value to go from $1000 to $500 if it depreciates 10% per year.

The value of r is r = 0.10 as its the decimal form of 10%

t is the unknown number of years we want to solve for

---------------------------

y = A(1 - r)^t

500 = 1000(1 - 0.1)^t

500 = 1000(0.9)^t

1000(0.9)^t = 500

0.9^t = 500/1000

0.9^t = 0.5

log( 0.9^t ) = log( 0.5 )

t*log( 0.9 ) = log( 0.5 )

t = log( 0.5 )/log( 0.9 )

t = 6.57881347896059

Note the use of logs to help us isolate the exponent.

HELP PLZ A circle inscribed in a triangle:

Answers

Answer:

The answer is the second photo.

Step-by-step explanation:

It's literally a circle in a triangle. So, it's the second one.

6(2+5)-2^2x2(9/3)+10/5

Answers

Answer:

The answer is 20

Step-by-step explanation:

i need help will rate you branliest

Answers

Answer D. the bottom one is the answer, because hyperbola is two curves that curve infinitely

Hope

Answer:

D. the bottom one is the answer, because hyperbola is two curves that curve infinitely

Triangles ABC and DEF are similar. Find the missing angles.

Answers

Angle A and D are similar
Angle B and E are similar
Angle C and F are similar

So this means they have congruent angles, meaning whichever angles are similar the angles will also be similar.

So since angle A is 62 degrees, angle D is 62 degrees too.

Since angle E is 80 degrees, angle B has to be 80 degrees as well.

All triangles add up to 180 degrees, so to find the angle measure of angle C and F, do:

180-(62+80)
180-142
= 38 degrees

So angles C and F are 38 degrees

Conclusion:
angles A and D: 62 degrees
angles B and E: 80 degrees
angles C and F: 38 degrees

Answer is D , others say it’s 64 but I got it wrong

Answers

Answer:

Oh no I am sorry! If you want answers to be done the real way let me know

Answer:I'm so sorry for you but congrats you did get the answer right it's just the test I guess

Step-by-step explanation:

2(3+ ‐ 4)(7‐3)÷(2‐ ‐2)​

Answers

the answer is 2 hopefully I answered your question in time


Helppppp thxxxxxxxxxx

Answers

Answer:

F. [tex] \frac{3}{2} [/tex]

Step-by-step explanation:

[tex] \frac{a + 2b}{b} = \frac{7}{2} [/tex]

Cross multiply:

7b= 2(a +2b)

Expand:

7b= 2a +4b

Bring all common variables to 1 side:

7b -4b= 2a

3b= 2a

divide by 2 on both sides:

[tex] \frac{3}{2} b = a[/tex]

divide by b on both sides:

[tex] \frac{3}{2} = \frac{a}{b} \\ \frac{a}{b} = \frac{3}{2} [/tex]

what is the area of rectangle

Answers

Answer:

Step-by-step explanation:

Find the area of a triangle whose two sides are 12 inches and 14 inches long, and has a perimeter of 34 inches. A. 23.24 in² B. 47.91 in² C. 24.74 in² D. 79.84 in²

Answers

Answer:

C

Step-by-step explanation:

That makes the third side 8 inches. Bc 34 -12 -14 = 8.

There's a formula that gives you the area of a triangle with all three sides.

It's kinda long so I won't write it out but it uses the semi-perimeter and it's called heron's formula if you want to search it up.

I just plugged the values into Heron's formula calculator.

The answer is around 47.906158268014, so C.

Area of the triangle is  [tex]47.91 \ in^{2}[/tex].

What is area of triangle?

The area of a triangle is defined as the total space occupied by the three sides of a triangle in a 2-dimensional plane.

Given

Perimeter  = 34

Two sides of the triangle = 12, 14

Third side of the triangle  = Perimeter - 12 - 14

= 34 - 12 - 14

= 8 inches

According to the figure, using Pythagoras theorem

[tex]h^{2}=8^{2} -x^{2}[/tex]

[tex]h^{2}=12^{2} - (14-x)^{2}[/tex]

[tex]8^{2} -x^{2} =12^{2} -(14-x)^{2}[/tex]

[tex]64-x^{2} =144-196+28x-x^{2}[/tex]

[tex]28x=116[/tex]

[tex]x=4.14[/tex]

So, [tex]h^{2} =8^{2} -4.14^{2}[/tex]

[tex]h^{2} =64-17.15[/tex]

[tex]h=6.844[/tex]

So, the area of the triangle  = [tex]\frac{1}{2} \times 14 \times 6.844[/tex]

= [tex]47.91 \ in^{2}[/tex]

Area of the triangle is  [tex]47.91 \ in^{2}[/tex].

Find out more information about area of the triangle here

https://brainly.com/question/22964077

#SPJ2


On dividing polynomial p(x) by a linear binomial, X - a, we get a quotien
statements must be proven true for the remainder theorem to be true

Answers

Answer:

Step-by-step explanation:

Hello, we can write

(1) p(x)=(x-a)q(x)+r

[tex]\boxed{\sf v}[/tex] True

It means that p(a)=0 * q(a) + r = r

so the first one is true.

[tex]\boxed{}[/tex] False

The second one is not to be proven true from the remainder theorem.

[tex]\boxed{\sf v}[/tex] True

For x different from a we can divide the equation (1) by (x-a).

[tex]\boxed{}[/tex] False

We cannot say anything on q(a).

[tex]\boxed{\sf v}[/tex] True

If the rest is 0 then it means that p(a) = 0

[tex]\boxed{\sf v}[/tex] True

If p(a) = 0 it means that the rest r = 0 and then p(x)=q(x)(x-a)

Thank you

The residents of a city voted on whether to raise property taxes. The ratio of yes votes to no votes was 6 to 5 . If there were 4545 no votes, what was the total number of votes?

Answers

Answer:

The total number of votes= 9999

Step-by-step explanation:

The ratio of vote specifically the ratio of yes to no vote in a city vote is 6 to 5.

There is a total of 4545 no votes.

Yes/no = 6/5

Yes= no(6/5)

Yes= 4545(6/5)

Yes= 5454

The total number of yes votes are 5454.

The total number of votes= yes votes+ no votes

The total number of votes= 5454+4545

The total number of votes= 9999

Integrate[Exp[Power[sinx,2]]sin2x,x]

Answers

Answer:

e^{sin²x}+c

Step-by-step explanation:

[tex]\int e^{sin^2x} sin 2x dx=?[/tex]

is this statement?

if so

then

[tex]put~sin^2x=t\\differentiate\\2 sin ~x~cos~x~dx=dt\\sin~2x ~dx=dt\\\int e^t~dt=e^t+c\\=e^{sin^2x}+c[/tex]

An initial deposit of $10000 is left in account for 5.5 years at 3.6% interest compounded continuously. What is the
final value of the account to the nearest dollar after the 5.5 years?

Answers

Answer:

11980

Step-by-step explanation:

This is simple interest.

We use the formula i=prt. Interest = price x rate x time

You just plug in your numbers, 10000 is your price, 5.5 years is the time, and 3.6% interest is rate.

Now our equation looks like this: i = 10000 x 5.5 x 0.036

(i converted the percent to a decimal)

So the interest is 1980, but the question asks for the final value, so we add the interest to the initial deposit, 1980 + 10000, and we get 11980 for our final value.

I apologize if my answer is wrong, but if it isn't I hope I helped :)

9514 1404 393

Answer:

  $12,190

Step-by-step explanation:

The formula for the balance in an account where continuous compounding occurs is ...

  A = Pe^(rt) . . . . where P is the principal invested at annual rate r for t years

  A = $10,000e^(0.036·5.5) = $10,000e^0.198

  A ≈ $12189.62 ≈ $12,190 . . . . rounded to the nearest dollar

I really need help please answer!

Answers

Answer:

-2, b, a+c

Step-by-step explanation:

Answer:

-2, b, a+c

Step-by-step explanation:

By looking at where A and C are on the number line, we can tell that A is a negative number close to zero and C is a positive number a little greater than four. This means that if we add the two together, we'll get a positive number a little below four.

By looking at the number line, we can tell that the value of B is a positive number a little below the number three.

Now that we know that B is less than A+C, and we know where -2 is on the number line (two marks to the left of zero) we can decide the least to greatest values.

Since negatives are always less than positives, we know that -2 has the smallest value. Next, we know that B is lower on the number line than A+C. So, in order, from least to greatest, the answer is:

-2, B, A+B

Hope this helps!! <3 :))

Other Questions
Given that 4i is a zero, factor the following polynomial function completely. Use the Conjugate Roots Theorem, if applicable. f(x)=x42x3+x232x240 Consider the geometric series modeling Pete's total and the formula for finding the sum of a series. Use thisinformation to determine the total amount of allowance Pete will be paid over the 16 weeks.Type the correct answer in the box. Use numerals instead of words.Over the span of 16 weeks, Pete will be paid a total of $in allowance. Ordena las letras de los enunciados siguientes siguiendo la estructura de los cuentos. m. Al conde le parce apropiado el consejo y lo pone en prctica con buenos resultados.s. Patronio establece una semejanza entre el caso que plantea el conde y un cuento.t. Patronio manifiesta y explica al conde el consejo que le ha pedido.h. Patronio narra el cuento(ejemplo).i. El conde Lucanor plantea un problema a Patronio, su servidor, y le pide que le aconseje.j. Don Juan Manuel resume la enseanza del cuento en dos versos pareados, a modo de moraleja. Corporations became a popular way to set up businesses because they:A.raise capital B.limit the risk for individual inventorsC.allow investors to share in the profits D.all of the choices are correct C no dolfneidlxnxbxjkdkdmd dnkfldmfkdBirifnff Since October 2008, the Federal Reserve has paid interest on excess reserves held by banks. Under these circumstances, if the Fed buys Treasury securities worth $300 million from a bank, how will the money supply be affected? Assume that the required reserve ratio is 10% and that all currency is deposited into the banking system. A. The money supply will increase by less than $3 billion, B. The money supply will increase by $3 billion C. The money supply will not change at all D. The money supply will increase by more than $3 billion can someone write a short paragraph responce to this? you need to fill a water jug with 8.23 L. you filled it with 2.92 L and then added 3.14 more how much more water do you need? 1.) Given 30 grams of CO2, how many moles of CO2 would you have? How many individual molecules of CO2 is that?2., Given 22 Grams of NaCl, How many moles of Nacl would you have? How many individual molecules of NaCl is that?3.) Given 50 grams of ammonia NH3, How many moles of NH3 would you have? How many Molecules of NH3 is that? 1. What are Regional Trading Blocs (RTBs)? Explain the 4 types of RTBs, making sure that the difference(s) between them is/are clearly stated as we move from one level to another. Discuss three (3) each of the advantages and disadvantages of the European Union to a named member country. You have been saving $12 each week for many weeks. One day, you decide to count your savings and find that you have $384. Write and solve a multiplication equation to find how many weeks w you have been saving. Write the equation for the horizontal line going through thepoint (3,7).y = what is the distance between the first and third quartiles of a data set called? Question #1: Select the correct answer. Choose the punctuation that correctly ends the sentence. Oh no, the house is on fire A. Question mark ( ? ) B. Exclamation point ( ! ) C. Period ( . ) Gabby is touring New York City with her family. They want to visit the Empire State Building, Central Park, and Times Square before their dinner reservations at 8:25 P.M. They want to spend 1 hour and 10 minutes at the Empire State Building, 2 hours and 50 minutes in Central Park, and 1 hour in Times Square. What is the latest time Gabby's family can start their tour in order to make it to dinner on time? The Lok Sabha is usually elected once every five years Pleaseee i'm struggling so hard on this Geometry thing, show work please A woman pulls on a 6.00-kg crate, which in turn is connected to a 4.00-kgcrate by a light rope. The light rope remains taut. Compared to the 6.00-kg crate,the lighter 4.00-kg cratePlease explain why any of these multiple choices is correct! Is ""I had a feeling that if I tried to play basketball today or do much of anything I might not survive."" a Simle Question 8 Multiple Choice During Mardi Gras and Ash Wednesday, a big celebration in Martinique, the inhabitants wear which specific colors?O Blue, white, blackO Pink, blue, redO Red, blue, whiteO Red, black, white